PT 48, sec. 3 ( RC), q.3,5 Forum

Prepare for the LSAT or discuss it with others in this forum.
Post Reply
caitriona

New
Posts: 10
Joined: Sat Nov 08, 2008 2:30 am

PT 48, sec. 3 ( RC), q.3,5

Post by caitriona » Sun May 08, 2011 9:03 pm

Hi,
I am struggling with some questions in RC,
why in q.3 right answer is E and not C, why in 5 right answer is C and not D?

Thank you.

SanDiegoJake

Bronze
Posts: 149
Joined: Mon Mar 07, 2011 3:17 pm

Re: PT 48, sec. 3 ( RC), q.3,5

Post by SanDiegoJake » Tue May 10, 2011 1:42 pm

As you surely noticed, each of these questions is phrased as , "author is most likely to agree...". While many erroneously read this question as an open-ended opportunity to guess what the author would agree with, the fact is that the correct answers must be directly supported by something in the passage to be correct. That's your issue right there - there is no direct support for your answer choices.

#3:
C) They did not create ANY art solely for aethetic purposes? That "any" is a very strong word, and you cannot show me a place in the passage where the author says this. Further, this answer choice incorrectly broadens the scope to all pre-historic hunter-gatherers, when the passage only concerns one such group - the Aurignacians.

E) Yes. Painting images to gain power is directly stated in lines 38-39. Additional cermonies are specifically mentioned in lines 49-50.

#5:
D) In the 1st paragraph, the author describes the theory that the art meant that they occupied a less-hostile environment (as only a less hostile environment could give the artists time to create such detailed art.). But the author refutes that theory in the following paragraphs. Author, in fact, says that it is unlikely that the paintings were creating such detailed art because they had extra time on their hands as a result of the less-hostile environment. Author goes on to explain the art was an important part of their food gathering - working the magic.

C) Yes, highly specialized roles of "dancer" and "shaman" are referenced in the final paragraph.

Bottom line: "Author most likely to agree" should be treated like an "inference" question, the answer choice must be directly supported by the passage.

Post Reply

Return to “LSAT Prep and Discussion Forum”